LSAT and Law School Admissions Forum

Get expert LSAT preparation and law school admissions advice from PowerScore Test Preparation.

 Administrator
PowerScore Staff
  • PowerScore Staff
  • Posts: 8917
  • Joined: Feb 02, 2011
|
#35430
Complete Question Explanation

(See the complete passage discussion here: lsat/viewtopic.php?t=14270)

GR, Must. The correct answer choice is (C)

This Global Reference, Must Be True question provides no information upon which to base a
prephrased answer, but the correct answer choice must pass the Fact Test, and must be verifiable
based on the information presented in the passage. Any choice that cannot be confirmed by the
passage should thus be ruled out of contention.

Answer choice (A): The author is doubtful that a trial could achieve true theoretical equipoise and
does not claim that such trials pose no ethical issues. This choice fails the Fact Test and should be
ruled out of contention.

Answer choice (B): The author does not mention the need to suspend any trials prematurely, so there
is no support in the passage for this choice, which thus cannot be the right answer.

Answer choice (C): This is the correct answer choice. A preference on the part of a participating
physician is “no ethical bar to participation in the trial,” the author says on line 53, confirming this to
be the right answer to this Must Be True question.

Answer choice (D): A trial that meets the standard of clinical equipoise would not necessarily meet
the much more restrictive standard of theoretical equipoise. Clinical equipoise is a different notion,
which is a more achievable standard that the author suggests in place of theoretical equipoise, which
the author believes would be nearly impossibly to achieve or maintain.

Answer choice (E): Although the author suggests clinical equipoise as the standard for comparative
clinical trials, this is not yet the general rule, so this choice fails the Fact Test and cannot be the right
answer.
 emilysnoddon
  • Posts: 64
  • Joined: Apr 22, 2016
|
#26168
For this question, I was choosing between answer choices B and C.I did not particularly like either answer choice for the following reasons

B) I felt was not necessarily true because it is never stated in the passage, rather the passage says that "If the standard of theoretical equipoise is adhered to, few comparative clinical trials could commence and even fewer could proceed to completion". The passage does not say that most trials adhere to the standard of theoretical equipoise though so I wasn't convinced by this answer. In addition, we are not sure that enough trials would get passed the commencement stage to warrant the inclusion of the word MOST.

C) I felt as though the part of the passage on "clinical equipoise" was entirely a suggestion based on the author's opinion, as documented by the inclusion of words like "should" and "can". For this reason, I thought that answer choice C was too strong in stating that it is not rendered unethical rather than saying it should not be considered unethical. If the answer choice had said should not be considered unethical I would have definitely chosen this answer choice.

I often feel as though in logical reasoning questions, discrepancies between the passage and the answer choice such as being an opinion or a fact cause answer choices to be wrong so I am confused about how to identify when I should take a statement as a fact or when I should be critical of it.

Any thoughts would be helpful.

Thanks!

Emily
 Shannon Parker
PowerScore Staff
  • PowerScore Staff
  • Posts: 147
  • Joined: Jun 08, 2016
|
#26285
Hi,

I understand the trouble you're having dealing with the "is"/"should" distinction. At least as far as this question goes, try to focus on the wording of the question, where it says that "according to the passage." While the notion of clinical equipoise may just be a suggestion of the author's, it is what makes up the passage, and therefore the only material you have to base your answer off of. Think of RC passages as complete works of fiction, where the only thing that exists is what the author writes.

Also, in this question, you can get further along by eliminating (B) as an option because of the reasoning that you laid out. The line "If the standard of theoretical equipoise is adhered to, few comparative clinical trials could commence..." which means that (B) could be true, but could also not be true, and therefore cannot be the answer. Having already eliminated the other choices, (C) is the only viable option left, and even if you were initially a little unsure, you could feel pretty confident about selecting it.

I hope this helps.

-Shannon
 graceli17
  • Posts: 8
  • Joined: Jun 27, 2017
|
#37331
Hi,

I'm still a bit confused on why B is wrong. I chose it because I thought "prematurely" has a similar meaning to "few comparative clinical trials could commence and even fewer could proceed to completion." If a researcher was biased towards one treatment from evidence at the start of a trial, under the theoretical equipose, the trial would have to be suspended.

Thank you!
 Luke Haqq
PowerScore Staff
  • PowerScore Staff
  • Posts: 742
  • Joined: Apr 26, 2012
|
#37603
Hi graceli17!

Answer (B) states, "Clinical researchers are often forced to suspend comparative clinical trials prematurely because initial data from the trials strongly favors one treatment over another."

The part you quoted starts in relevant part around line 25, "Even if researchers judged the evidence to be balanced at the start of a comparative clinical trial, such a balance would be extremely fragile, liable to be “tipped” by small accretions of evidence as the study progresses. Consequently, if the standard of theoretical equipoise is adhered to, few comparative clinical trials could commence and even fewer could proceed to completion."

I think that critical point to seeing why (B) is wrong is to note that the italicized part only follows "if the standard of theoretical equipoise is adhered to." Since this is a must-be-true question, (B) can't be right because we don't know that "Clinical researchers are often forced to suspend comparative clinical trials." Another possibility is that the standards of theoretical equipoise are not adhered to.
 harvoolio
  • Posts: 63
  • Joined: Apr 25, 2018
|
#45635
I answered (D) quickly and mixed things up. At of curiosity would (D) have been correct if it read instead "A comparative clinical trial that meets the standard of theoretical equipoise would therefore also meet the standard of clinical equipoise. In other words if a physician felt equally about each treatment option (theoretical equipoise) by definition there would be a medical expert for each (or against each) and hence qualify for clinical equipoise - or does the passage not even support this?

Thanks.
 James Finch
PowerScore Staff
  • PowerScore Staff
  • Posts: 943
  • Joined: Sep 06, 2017
|
#46262
Hi Harvoolio,

Yes, that would make sense based on the way the passage has described both clinical and theoretical equipoise. The passage is arguing for the adoption of a more realistic clinical equipoise, in which researchers may have treatment preferences but acknowledge that others may reasonably disagree with them. Theoretical equipoise eliminates the preferences, meaning the researchers would consider both options equal before performing a trial, which would mean that they would acknowledge the reasonableness of recommending either as a course of treatment (satisfying clinical equipoise requirements.)
 tetsuya0129
  • Posts: 73
  • Joined: Jun 20, 2018
|
#85767
Hi PowerScore Staff,

"I think that critical point to seeing why (B) is wrong is to note that the italicized part only follows "if the standard of theoretical equipoise is adhered to." {...} (B) can't be right because we don't know that "Clinical researchers are often forced to suspend comparative clinical trials." Another possibility is that the standards of theoretical equipoise are not adhered to."

I thought the theoretical equipoise was typically adhered to because of Line 15-16 "[...] the conception of equipoise that is typically employed [...] may be too strict." Or, a typically-employed conception doesn't have to be typically adhered to, does it?

Thank you for the help.
Leon
User avatar
 ToadKing
  • Posts: 18
  • Joined: Jan 17, 2021
|
#87048
For (C), I thought that the answer was true for clinical equipoise but not for theoretical equipoise. Should I think of "According to the passage" questions as "According to the author"? This is the only way I could reach (C) as a correct answer. Otherwise, the negation of (C) (it is rendered unethical) would be correct because it would be supported by the paragraphs about theoretical equipoise.
 Robert Carroll
PowerScore Staff
  • PowerScore Staff
  • Posts: 1787
  • Joined: Dec 06, 2013
|
#87570
Toad,

I can't think of there being a distinction between the passage and the author's viewpoint. It's not as if the passage is an objective viewpoint and the author has a viewpoint within that - the entire passage is just what the author says. So "according to the passage" and "according to the author" should coincide.

Robert Carroll

Get the most out of your LSAT Prep Plus subscription.

Analyze and track your performance with our Testing and Analytics Package.